Difference between revisions of "2025 AIME II Problems/Problem 7"

(Solution 1)
Line 24: Line 24:
  
 
~Mitsuihisashi14
 
~Mitsuihisashi14
 +
 +
==Solution 2==
 +
We take the complement and use PIE. Suppose the LCM of the elements of the set is NOT <math>2025</math>. Since <math>2025=3^4 \cdot 5^2</math>, it must be that no element <math>x</math> in the subset satisfies <math>v_3(x)=4</math> OR no element <math>x</math> in the subset satisfies <math>v_5(x)=2</math> (in this case, <math>v_p(n)</math> gives the exponent of <math>p</math> in the prime factorization of <math>n</math>). For the first case, there are <math>4 \cdot 3 = 12</math> possible divisors that could be in our subset (<math>v_3(x)=0,1,2,3,v_5(x)=0,1,2</math> are possible), for a total count of <math>2^{12}</math> subsets. In the second case, there are <math>5 \cdot 2 = 10</math> possible divisors that could be in our subset, for a total count of <math>2^{10}</math> subsets. However, if both conditions are violated, then there are <math>4 \cdot 2 = 8</math> divisors that could be in our subset, for a total count of <math>2^8</math> subsets. Hence, by PIE, the number of subsets whose LCM is NOT <math>2025</math> is equal to <math>2^{12}+2^{10}-2^8</math>. The answer is then <cmath>1-\frac{2^{12}+2^{10}-2^8}{2^{5 \cdot 3}}=\frac{109}{128} \implies \boxed{237}.</cmath>
 +
 +
~cxsmi

Revision as of 23:00, 13 February 2025

Problem

Let $A$ be the set of positive integer divisors of $2025$. Let $B$ be a randomly selected subset of $A$. The probability that $B$ is a nonempty set with the property that the least common multiple of its element is $2025$ is $\frac{m}{n}$, where $m$ and $n$ are relatively prime positive integers. Find $m+n$.

Solution 1

We split into different conditions:

Note that the set needs to satisfy with all of the elements' lcm = 2025, we need to ensure that the set has at least 1 number that is a multiple of 3^4 and a number that is a multiple of 5^2.

Multiples of 3^4: 81, 405, 2025 Multiples of 5^2: 25, 75, 225, 675, 2025

If the set B contains 2025, then all of the rest 14 factors is no longer important. The valid cases are 2^14.

If the set B doesn't contain 2025, but contains 405, we just need another multiple of 5^2. It could be 1 of them, 2 of them, 3 of them or 4 of them, which has 2^4 - 1 = 15 cases. Excluding 2025, 405, 25, 75, 225, 675, the rest 9 numbers could appear or not appear. Therefore, this case has a valid case of 15 * 2^9.

If set B doesn't contain 2025 nor 405, it must contain 81. It also needs to contain at least 1 of the multiples from 5^2, where it would be 15 * 2^8.

The total valid cases are 2^14 + 15 * (2^9 + 2^8), and the total cases are 2^15. The answer is 2^8 * (64 + 30 + 15) / 2^8 * 2^7 = 109/128. Desired answer: 109 + 128 = 237

(Feel free to edit any latex or format problems)

~Mitsuihisashi14

Solution 2

We take the complement and use PIE. Suppose the LCM of the elements of the set is NOT $2025$. Since $2025=3^4 \cdot 5^2$, it must be that no element $x$ in the subset satisfies $v_3(x)=4$ OR no element $x$ in the subset satisfies $v_5(x)=2$ (in this case, $v_p(n)$ gives the exponent of $p$ in the prime factorization of $n$). For the first case, there are $4 \cdot 3 = 12$ possible divisors that could be in our subset ($v_3(x)=0,1,2,3,v_5(x)=0,1,2$ are possible), for a total count of $2^{12}$ subsets. In the second case, there are $5 \cdot 2 = 10$ possible divisors that could be in our subset, for a total count of $2^{10}$ subsets. However, if both conditions are violated, then there are $4 \cdot 2 = 8$ divisors that could be in our subset, for a total count of $2^8$ subsets. Hence, by PIE, the number of subsets whose LCM is NOT $2025$ is equal to $2^{12}+2^{10}-2^8$. The answer is then \[1-\frac{2^{12}+2^{10}-2^8}{2^{5 \cdot 3}}=\frac{109}{128} \implies \boxed{237}.\]

~cxsmi